LSAT and Law School Admissions Forum

Get expert LSAT preparation and law school admissions advice from PowerScore Test Preparation.

User avatar
 Dave Killoran
PowerScore Staff
  • PowerScore Staff
  • Posts: 5852
  • Joined: Mar 25, 2011
|
#88521
Complete Question Explanation
(The complete setup for this game can be found here: lsat/viewtopic.php?f=180&p=40792#p40792)

The correct answer choice is (E).

The condition in the question stem, plus the application of the second rule, establishes the following initial setup:

G2-Q9-d1.png

The three remaining students are J, K, and S. Because J and K cannot be accompanied by the same adult, the must be split between O and P. Via Hurdling the Uncertainty, then, S must be accompanied by P:

G2-Q9-d2.png

Accordingly, answer choice (E) must be true and is correct.
You do not have the required permissions to view the files attached to this post.

Get the most out of your LSAT Prep Plus subscription.

Analyze and track your performance with our Testing and Analytics Package.